Understanding epsilon-delta def of limits

  • Thread starter Thread starter foxjwill
  • Start date Start date
  • Tags Tags
    Limits
Click For Summary
The discussion focuses on understanding the epsilon-delta definition of limits, particularly in disproving that the limit of x^2 as x approaches 1 equals 2. To demonstrate this, one can set an interval around x=1 and analyze the corresponding epsilon values. When delta is large, the condition holds, but as delta decreases, it becomes evident that the limit does not satisfy the epsilon condition for all cases. This indicates that the limit of x^2 as x approaches 1 is not equal to 2. The conversation emphasizes the importance of carefully choosing delta and epsilon to validate or invalidate limit statements.
foxjwill
Messages
350
Reaction score
0

Homework Statement


I'm having trouble conceptually understanding the epsilon-delta definition of limits. How do you use it to disprove a limit? For example, how would you use it to show that \lim_{x \to 1} x^2 \neq 2?


Homework Equations





The Attempt at a Solution

 
Physics news on Phys.org
imagine you've got the function x plotted in a graph(you can actually draw it in a piece of paper). and you want to show that <br /> \lim_{x \to 1} x \neq 2. First, set an interval ]1-delta,1+delta[ in x axes, see the image of the 1+delta; epsilon will be=|f(1+delta)-f(1)|. (in general, it will be the max{|f(1+delta)-f(1)|,|f(1-delta)-f(1)|}, in this case, the function is similar to the right and left of f(1), so, only one part is needed because both are equal).
Now, you see that if the delta is very big, say 4, then epsilon=4 rigth?! so, (the second part of the condition, the epsilon part) is |2-f(1)|=2-1=1<epsilon: the condition is verified.(2 because you are testing the condition on the point 2 you are asking: "is it really true that the condition is verified to EVERY delta?")

Note however that if you put delta lower, say 0.5, epsilon is 0.5 and |2-f(1)|=|2-1|>epsilon:. Exists a delta that don't verify the condition, that implies that isn't true that for every delta, exists..bla bla bla...so, the limit isn't 2.

Hope this helps solve your problem...
and hope i didn't make any mistake...^_^
 
Last edited:
Perhaps in this way
can Be clear idea

[url=http://www.up07.com/up7][PLAIN]http://www.up07.com/up7/uploads/6225e0aea0.jpg[/url][/PLAIN]
 
m_s_a said:
Perhaps in this way
can Be clear idea

[url=http://www.up07.com/up7][PLAIN]http://www.up07.com/up7/uploads/6225e0aea0.jpg[/url][/PLAIN]

but how do you put that into an epsilon-delta proof?
 
[url=http://www.up07.com/up7][PLAIN]http://www.up07.com/up7/uploads/df5040ccd8.jpg[/url][/PLAIN]
[url=http://www.up07.com/up7][PLAIN]http://www.up07.com/up7/uploads/4c7f5603e7.jpg[/url][/PLAIN]

No requirement that the value of f(x0)

The smaller values for delta & epsilon Leads to ...what?
 
Question: A clock's minute hand has length 4 and its hour hand has length 3. What is the distance between the tips at the moment when it is increasing most rapidly?(Putnam Exam Question) Answer: Making assumption that both the hands moves at constant angular velocities, the answer is ## \sqrt{7} .## But don't you think this assumption is somewhat doubtful and wrong?

Similar threads

  • · Replies 31 ·
2
Replies
31
Views
2K
  • · Replies 13 ·
Replies
13
Views
4K
  • · Replies 9 ·
Replies
9
Views
4K
  • · Replies 2 ·
Replies
2
Views
1K
  • · Replies 6 ·
Replies
6
Views
2K
  • · Replies 9 ·
Replies
9
Views
3K
  • · Replies 1 ·
Replies
1
Views
1K
  • · Replies 3 ·
Replies
3
Views
2K
  • · Replies 8 ·
Replies
8
Views
2K
  • · Replies 10 ·
Replies
10
Views
2K